\item \ifassignmentsheet \point{1} \else \prac \fi Consider the sentence $\phi = \exists x \forall y (P(x,y) \implies (Q(x,y) \lor R(x,y)))$. Does the following model satisfy $\phi$? The model $M$ consists of: \begin{itemize} \item $A = \{a, b, c\} $ \item $P^{M} = \{(a,a), (a,b), (b,a), (b,b), (c,a), (c,b)\}$ \item $Q^{M} = \{(a,m) | m \in A \}$ \item $R^{M} = \{(a,a), (b,a), (a,c), (b,c), (c,c)\}$ \end{itemize} % \item The model $M'$ consists of: %\begin{itemize} %\item $A = \mathbb{Z} $ %\item $P^{M} = \{(m,n) | m \ge n \}$ %\item $Q^{M} = \{(m,n) | m \le n \}$ %\item $R^{M} = \{(m,n) | m = -n \}$ %\end{itemize}